How to Approach Solving Set Theory Equalities

Click For Summary
To approach set theory equalities, one effective method is to prove that both sides of the equation are subsets of each other. For example, to prove (A u B) n (C u D) = (A n (C u D)) u (B n (C u D)), start by letting x be an element of the left-hand side and demonstrate that it also belongs to the right-hand side. This involves showing that if x is in (A u B) and (C u D), it can be expressed as being in either A or B and either C or D. After establishing this direction, the reverse must also be shown to complete the proof. This systematic approach ensures clarity in proving set equalities.
racshot65
Messages
7
Reaction score
0
Hi,

I'm struggling to understand how to approach set theory equality questions for example:

True or false?

(A n B) is a subset of (A u B)


Is quite simple as its obvious the intersection will contain everything that is in the union

But what about a more complex question like ...


True or false

(A u B) n (C u D) = (A n (C u D)) u (B n (C u D))


There must be some method you follow to tackle a question like this as it isn't obvious like the previous question ?


My question is what is the method ?


Thanks
 
Physics news on Phys.org
In general, if you have something like this:

Prove: S_1 = S_2 then you show that S_1 \subset S_2 and S_2 \subset S_1. Then the equality follows. In the above example, you would say, let x \in (A \cup B) \cap (C \cup D) now you have to show that x is an element of the RHS of your equation. Then you "just" do this in the opposite direction. So, you know that x is in either A or B AND it is in either C or D. Now, it should be clear (with a little work) that (A\cup B) \cap (C \cup D) \subset (A \cap (C \cup D)) \cup (B \cap (C \cup D)). Now just show the other direction.
 
Last edited:
Question: A clock's minute hand has length 4 and its hour hand has length 3. What is the distance between the tips at the moment when it is increasing most rapidly?(Putnam Exam Question) Answer: Making assumption that both the hands moves at constant angular velocities, the answer is ## \sqrt{7} .## But don't you think this assumption is somewhat doubtful and wrong?

Similar threads

  • · Replies 12 ·
Replies
12
Views
2K
Replies
1
Views
2K
  • · Replies 1 ·
Replies
1
Views
2K
  • · Replies 2 ·
Replies
2
Views
1K
  • · Replies 6 ·
Replies
6
Views
2K
  • · Replies 8 ·
Replies
8
Views
2K
  • · Replies 7 ·
Replies
7
Views
2K
  • · Replies 9 ·
Replies
9
Views
6K
  • · Replies 3 ·
Replies
3
Views
1K
  • · Replies 2 ·
Replies
2
Views
1K